CT Exam Five Questions

Pataasin ang iyong marka sa homework at exams ngayon gamit ang Quizwiz!

*The ideal antibiotic duration range, depending on the selected antibiotic, in patients with uncomplicated UTIs is: A. 1-5 days B. 7 days C. 10 days D. 14 days E. 21 days

A. 1-5 days

*Which patient is most likely to have acute bacterial rhinosinusitis? A. 18-year-old woman with rhinorrhea, cough, headache, and purulent nasal discharge for 10 days and tooth pain for 2 days B. 42-year-old man with sneezing, nasal congestion, and purulent nasal discharge for 5 days C. 35-year-old woman with sore throat for 3 days and sneezing, nasal congestion, and cough for 5 days D. 55-year-old man with sneezing and postnasal discharge for 5 days with headache and cough for the last 3 of those 5 days

A. 18-year-old woman with rhinorrhea, cough, headache, and purulent nasal discharge for 10 days and tooth pain for 2 days

Patients with subclinical or mild hypothyroidism or hyperthyroidism will have: A. Abnormal TSH and normal free T4 (FT4) levels B. Abnormal TSH and abnormal FT4 levels C. Normal TSH and abnormal FT4 levels D. Normal TSH and normal FT4 levels E. Abnormal TSH and abnormal free triiodothyronine (FT3) levels

A. Abnormal TSH and normal free T4 (FT4) levels

BK is a 72-year-old woman with a history of a vertebral compression fracture and a DXA T-score of -2.7 in the left hip. What is the best initial therapy for this patient? A. Alendronate 70 mg orally once weekly B. Zoledronic acid 5 mg IV yearly C. Calcitonin nasal spray 1 spray daily in alternating nostrils D. Conjugated estrogen/medroxyprogesterone 0.625/2.5 mg orally daily E. Denosumab 60 mg subcutaneously every 6 months

A. Alendronate 70 mg orally once weekly

A 39-year-old woman presents to an ambulatory care clinic with a 10-day history of nasal congestion and postnasal discharge. She tried oral phenylephrine for the last 2 days with very little relief of her congestion. Over the last 2 days, she developed right maxillary facial pain, a fever of 100.4?F (38.0 °C), and a cough that is most pronounced in the morning. She is allergic to penicillin ("abdominal pain and diarrhea") and she has not been treated with any antibiotics for the past 5 years. What is the most appropriate antibiotic for this patient? A. Amoxicillin for 10 days B. Moxifloxacin for 10 days C. Azithromycin for 3 days D. Cefdinir for 5 days

A. Amoxicillin for 10 days

*Which of the following antibiotic regimens would be the most appropriate for a patient with viridans group streptococcal IE (penicillin MIC greater than 0.5 mcg/mL [greater than 0.5 mg/L])? A. Ampicillin B. Penicillin G C. Cefazolin D. Ceftriaxone

A. Ampicillin

*Which of the following is the best synergistic combination for treatment of a native-valve enterococcal endocarditis caused by strains susceptible to penicillin, aminoglycosides, and vancomycin? A. Ampicillin plus gentamicin B. Ampicillin plus tobramycin C. Ampicillin plus rifampin D. Vancomycin plus rifampin

A. Ampicillin plus gentamicin

Which biologic DMARD is recommended in the treatment guidelines for juvenile idiopathic arthritis and not in the treatment guidelines for rheumatoid arthritis? A. Anakinra B. Abatacept C. Rituximab D. Infliximab E. Tocilizumab

A. Anakinra

*Adjunctive therapies for acute otitis media that are effective in reducing symptoms include: A. Antipyretics/analgesics B. Antihistamines C. Decongestants D. None of the above

A. Antipyretics/analgesics

A 36yearold female presents with a 3day history of high fevers, headaches, and bloody diarrhea. She also reports fairly significant abdominal pain and states that she thinks she might have appendicitis. She reports eating some chicken that may have been undercooked a few days previously. She also takes a protonpump inhibitor on a daily basis for gastroesophageal reflux disease. Which one of the following should be recommended for the treatment of this patient: A. Azithromycin B. Levofloxacin C. Metronidazole D. Trimethoprimsulfamethoxazole E. No antibiotic therapy is indicated for this infection.

A. Azithromycin

Which of the following statements regarding treatment considerations is false? A. Bacteriostatic agents with synergistic activity are preferred. B. Dosage of antimicrobial is typically the traditional amount. C. Parenteral antimicrobials are typically required for treatment. D. Penetration into the vegetation is necessary to achieve adequate killing.

A. Bacteriostatic agents with synergistic activity are preferred.

An acceptable initial IP empiric antimicrobial therapy for a 35yearold with spontaneous bacterial peritonitis and a history of rash with cephalexin is: A.Cefazolin plus ceftazidime (LD 500 mg/L, MD 125 mg/L for each) B. Tobramycin (LD 8 mg/L, MD 4 mg/L) C. Vancomycin (LD 1,000 mg/L, MD 25 mg/L) D.Imipenemcilastin (LD 250 mg/L, MD 50 mg/L)

A. Cefazolin plus ceftazidime (LD 500 mg/L, MD 125 mg/L for each)

Which of the following drugs is contraindicated in pregnant women who have been diagnosed with gonorrhea? A. Ciprofloxacin B. Ceftriaxone C. Azithromycin D. Cefixime

A. Ciprofloxacin

Which of the following factors is/are the most important considerations for determining the duration of therapy for DR? A. Clinical pulmonary infection score B. Time to the start of clinical improvement C. Risks for toxicity associated with the therapy D. A and C E. All of the above

A. Clinical pulmonary infection score

*Before an empirical regimen can be selected, which of the following must be evaluated? A. Comorbid conditions that the patient has that could influence the regimen or outcomes B. What drugs cover all of the organisms that could cause pneumonia (no matter how rare the incidence is) C. The patient's cardiac function D. A and C E. All of the above

A. Comorbid conditions that the patient has that could influence the regimen or outcomes

*PW is a 53-year-old woman with a history of esophageal stricture status postdilatation due to longstanding GERD and vasomotor symptoms associated with menopause. A recent DXA scan showed a T-score of -2.5 in the lumbar spine and -2.6 in the left hip. Which therapy is most appropriate for PW? A. Denosumab 60 mg subcutaneously every 6 months B. Raloxifene 60 mg orally once daily C. Risedronate 35 mg orally once weekly D. Ibandronate 150 mg orally once monthly E. Calcitonin 200 IU intranasal daily in alternating nostrils

A. Denosumab 60 mg subcutaneously every 6 months

MM is an 82-year-old frail woman who is recovering from a vertebral compression fracture. At her follow-up appointment today, her primary care provider has asked the team to make interventions to reduce her fall risk. Which factor would decrease her risk of falls? A. Discontinuation of oxybutynin B. Changing amitriptyline to diphenhydramine for sleep C. Use of lorazepam several nights per week D. Initiation of metoprolol E. Tapering of nonsteroidal anti-inflammatory agents

A. Discontinuation of oxybutynin

Which one of the following disorders is a possible extraintestinal manifestation of inflammatory bowel disease? A. Erythema nodosum B. Migraine headaches C. Asthma D. Sinusitis

A. Erythema nodosum

CC is a 24-year-old female inquiring about the receiving the influenza and pneumococcal vaccines at your pharmacy. She has asthma, has no drug or food allergies, and is 6 months pregnant. Which of the following vaccines can she receive? A. Fluzone intradermal B. Influenza intranasal C. 13-valent conjugated pneumococcal D. 7-valent conjugated pneumococcal E. None, she has to wait until after the pregnancy to be vaccinated

A. Fluzone intradermal

A 55-year old female with pan-susceptible E. coli pyelonephritis who is discharged from the hospital can be treated will any of the following antibiotic regimens in the outpatient setting except: A. Fosfomycin B. Trimethoprim-sulfamethoxazole C. Ciprofloxacin D. Amoxicillin-clavulanic acid E. Levofloxacin

A. Fosfomycin

Genital warts are caused by which of the following? A. Human papillomavirus B. Herpes simplex virus C. Cytomegalovirus D. Haemophilus ducreyi

A. Human papillomavirus

*T lymphocytes may influence the pathogenesis of RA by: I. Releasing cytokines II. Releasing antibodies III. Presenting antigens to B lymphocytes A. I only B. III only C. I and II D. II and III E. I, II, and III

A. I only

For the patient in the previous question, which of the following is the most appropriate treatment? A. Ibuprofen 90 mg orally every 8 hours B. Methotrexate 5 mg orally once weekly C. Etanercept 25 mg IV once weekly D. Prednisone 5 mg orally daily E. Abatacept 500 mg IV on days 1 and 15 and then every 28 days thereafter

A. Ibuprofen 90 mg orally every 8 hours

Treatment of bacterial vaginosis includes: A. Intravaginal use of clindamycin or metronidazole B. Use of metronidazole in the first trimester of pregnancy to avoid transmission C. Counseling the patient to avoid alcohol consumption until the third dose D. Daily application of metronidazole for 30 days

A. Intravaginal use of clindamycin or metronidazole

*Which statement is true concerning Gardisil? A. It protects against HPV genotypes 6, 11, 17, and 18. B. It is approved for the treatment of males only ages 13 to 26 years. C. t is used to treat breast cancer. D. Vaccine is given in four injections over a period of 9 months.

A. It protects against HPV genotypes 6, 11, 17, and 18.

*Which of the following peripheral manifestations causes painless plaques when it occurs? A. Janeway lesions B. Osler nodes C. Petechiae D. Splinter hemorrhage

A. Janeway lesions

Two days after disembarking from a cruise ship, a 26yearold female presents to her local medical clinic complaining of abdominal cramping and vomiting for the past 36 hours. She states that she noted some watery diarrhea that developed the last day or so of her cruise, but now the stools are of smaller volume, but appear to be bloody. A presumptive diagnosis of shigellosis is made. Which one of the following therapies should be recommended: A. Levofloxacin B. Loperamide C. Metronidazole D. Trimethoprimsulfamethoxazole E. No antibiotic therapy is indicated for this infection.

A. Levofloxacin

Nitrofurantoin or fosfomycin are currently recommended over ciprofloxacin as first-line empiric agents for treatment of uncomplicated cystitis because these two agents possess: A. Limited scope of activity directed toward common uropathogens B. Increased adherence potential C. Greater activity against E. coli D. Lower probability for resistance emergence in E. coli E. No differences have been shown between fluoroquinolone, nitrofurantoin, and fosfomycin treatment regimens

A. Limited scope of activity directed toward common uropathogens

ML is a 65-year-old Asian man who presents with symptoms of tremor, slowness, and stiffness. His current medications include loxapine, atorvastatin, hydrochlorothiazide, and clopidogrel. Which of the following medications could contribute to his PD symptoms? A. Loxapine B. Atorvastatin C. Hydrochlorothiazide D. A and B E. None of the above

A. Loxapine

Which of the following is considered the drug of choice for trichomoniasis? A. Metronidazole B. Ceftriaxone C. Azithromycin D. Doxycycline

A. Metronidazole

DR underwent bronchoscopy, and the Gram stain of the bronchoalveolar lavage shows moderate gram-positive cocci in pairs, moderate gram-negative bacilli, and many WBCs. Which of the following would be the most appropriate empirical regimen for DR's pneumonia? A. Moxifloxacin 400 mg IV or PO every 24 hours B. Nafcillin 2 g IV every 6 hours plus ceftriaxone 1 g IV every 24 hours plus tobramycin C. Vancomycin 1,500 mg IV every 12 hours plus ceftriaxone 1 g IV every 24 hours plus tobramycin D. Azithromycin 500 mg IV every 24 hours plus ceftriaxone 1 g IV every 24 hours E. Ertapenem 1 g IV every 24 hours plus ceftriaxone 1 g IV every 24 hours

A. Moxifloxacin 400 mg IV or PO every 24 hours

*PF is a 50-year-old male who smokes two packs of cigarettes per day. Which of the following host defenses that protect the lung are known to be impaired by the smoke? A. Mucous and ciliated cells B. Alveolar macrophages C. Immunoglobulin (IgA, IgG, and IgM) D. A and C E. All of the above

A. Mucous and ciliated cells

Which one of the following counseling points should be discussed with a patient regarding a new prescription for methotrexate for RA? A. New onset cough B. An infusion reaction C. Changes in peripheral vision D. Yellow-orange skin discoloration E. Increased patchy skin pigmentation

A. New onset cough

*The preferred treatment for an asymptomatic, 31-year-old pregnant woman at 36 weeks' gestation with a routine clean-catch urine growing E. coli is: A. Nitrofurantoin B. Trimethoprim-sulfamethoxazole C. Ciprofloxacin D. Doxycycline E. No therapy required for asymptomatic bacteruria

A. Nitrofurantoin

Cross-allergenicity between penicillins and cephalosporins: A. Occurs at a low rate, less than 6% B. Is uncommon, less than 0.05% C. Is common, greater than 90% D. Is variable but most studies estimate around 50% E. Never occurs

A. Occurs at a low rate, less than 6%

A 12-year-old girl presents to her pediatrician's office with a 2-day history of throat pain, diminished appetite, fever of 100.3?F (37.9°C), and fatigue. Her mother reports that "strep throat" has been identified at her school recently. Physical examination reveals pharyngeal and tonsillar erythema without exudates. What is the most appropriate course of action? A. Perform a rapid streptococcal antigen detection test and treat with antibiotics if the test is positive B. Perform a throat culture and immediately initiate antibiotics because this patient is likely to have streptococcal pharyngitis C. Initiate antibiotics without any diagnostic testing since her symptoms and possible exposure to streptococcal disease are sufficient to make the diagnosis D. Diagnose her with a viral upper respiratory tract infection and send her home without performing any diagnostic testing

A. Perform a rapid streptococcal antigen detection test and treat with antibiotics if the test is positive

Which medication should not be used for maintenance therapy of CD? A. Prednisone B. Azathioprine C. Mesalamine D. Infliximab

A. Prednisone

*A solitary, painless chancre is most consistent with which of the following stages of syphilis? A. Primary B. Secondary C. Latent early D. Latent late

A. Primary

*Which one of the following is the most appropriate treatment approach in a patient at risk for developing OA? A. Promote lifestyle changes targeted at risk factors for OA B. Initiate acetaminophen 325 mg every 4 to 6 hours as needed C. Initiate ibuprofen 200 mg every 6 hours as needed D. Initiate acetaminophen 325 mg every 4 to 6 hours as needed, and promote lifestyle changes targeted at risk factors for OA E. Initiate ibuprofen 200 mg every 6 hours as needed, and promote lifestyle changes targeted at risk factors for OA

A. Promote lifestyle changes targeted at risk factors for OA

MC is a 38-year-old healthy woman who presents to your booth at a health fair. She asks what she can do to maintain bone health. Her mother had severe osteoporosis with vertebral compression fractures, spinal kyphosis, hip fracture, and limited mobility. She is 64 in (163 cm) tall and weighs 52.2 kg (115 lb). She is premenopausal. Her medications include ethinyl estradiol/norethindrone daily and a multivitamin daily. Her peripheral DXA of the heel shows a T-score of -1.3 and a Z-score of -0.8. Which exercise regimen recommendation is appropriate for MC to maintain or improve bone health? A. Stair climbing B. Bicycle riding C. Water aerobics D. Yoga E. Spin class

A. Stair climbing

A person taking oxycodone 5 mg/acetaminophen 325 mg, 2 tablets every 4 hours as needed for pain, starts itching all over within 20 minutes of each dose. Which of the following statements is true? A. The itching is most likely due to the oxycodone stimulating mast cell release. B. The itching is most likely due to a Gell and Coombs type I reaction. C. The itching is a warning that the patient is allergic to opiates and should not receive them. D. The itching is most likely due to the underlying cause of the patient's pain. E. The itching is an early sign of anaphylaxis and the patient should receive epinephrine immediately.

A. The itching is most likely due to the oxycodone stimulating mast cell release.

AK is a 12-year-old child who will have to undergo a splenectomy. In order to minimize pneumococcal disease after the splenectomy, AK should receive: A. The polysaccharide vaccine B. The 13-valent conjugated vaccine C. No vaccine because they are not effective in the absence of a spleen and AK should be placed on penicillin prophylaxis to prevent pneumococcal disease D. All of the above E. None of the above

A. The polysaccharide vaccine

Prior to initiating therapy with azathioprine, patients should be evaluated for activity of which enzyme?? A. Thiopurine methyltransferase B. Azathioprine dehydrogenase C. Dihydrofolatereductasesecretory diarrhea D. Alanine aspartase

A. Thiopurine methyltransferase

*Loperamide should be avoided in patients with active UC due to its ability to induce which one of the following complications? A. Toxic megacolon B. Intestinal stricture C. Fistula formation D. Pancreatitis

A. Toxic megacolon

*Which one of the following is characteristic of osteoarthritis (OA)? A. Unilateral joint pain upon waking B. Joint stiffness in the morning lasting 45 minutes C. An elevated C-reactive protein (CRP) level D. An elevated erythrocyte sedimentation rate (ESR) E. Upper extremities are most susceptible

A. Unilateral joint pain upon waking

Which statement is correct concerning treatment for HSV? A. Valacyclovir can be used to treat sexual transmission of HSV to an uninfected partner. B. Once infected with HSV, antibodies prevent reactivation. C. If a patient is resistant to acyclovir, the strains are susceptible to famciclovir. D. Because of vaccinations, HSV cannot be transmitted during pregnancy.

A. Valacyclovir can be used to treat sexual transmission of HSV to an uninfected partner.

*Pick the symptom that is least characteristic of allergic rhinitis. A. dermatitis B. sneezing C. nasal congestion D. rhinorrhea E. nasal itch

A. dermatitis

Pick the symptom that is least characteristic of allergic rhinitis. A. dermatitis B. sneezing C. nasal congestion D. rhinorrhea E. nasal itch

A. dermatitis

A 24-year-old female, who is known to be 8 weeks pregnant, has been on oral montelukast, oral loratadine, and intranasal cromolyn for moderately severe, persistent allergic rhinitis. Her symptoms of nasal congestion, rhinorrhea, sneezing, and ocular itching are still poorly controlled, despite good adherence. Pick the most appropriate drug from the choices below to recommend as an additional and/or replacement medication. A. intranasal budesonide B. oral diphenhydramine C. intranasal phenylephrine D. oral pseudoephedrine E. intranasal ipratropium

A. intranasal budesonide

Pick the most appropriate drug from the choices below, to recommend as an OTC medication for initial therapy of mild, intermittent (and seasonal) allergic rhinitis in an otherwise healthy 20-year-old female who is sexually active, has irregular menstrual periods and does not remember when her last menses started. A. oral fexofenadine B. intranasal azelastine C. oral levocetirizine D. intranasal ipratropium E. oral pseudoephedrine

A. oral fexofenadine

*The two common drug classes most likely to cause anaphylaxis are: A. ß-Lactam antibiotics and aspirin/nonsteroidal antiinflammatory drugs B. ß-Lactam antibiotics and opiates C. Opiates and aspirin/nonsteroidal anti-inflammatory drugs D. Opiates and anticonvulsants E. ß-Lactam antibiotics and anticonvulsants

A. ß-Lactam antibiotics and aspirin/nonsteroidal antiinflammatory drugs

A 65-year-old man undergoing radiation therapy for locally metastatic prostate cancer complains of the abrupt onset of a severe headache associated with blurred vision. He has no history of a headache disorder, and his pain is unrelieved by nonsteroidal antiinflammatory drugs (NSAIDs. What is the most appropriate next step in his evaluation? A.CT imaging of the brain B. Snellen test for visual acuity C. Headache diary D. Lumbar puncture

A.CT imaging of the brain

A 59yearold is undergoing peritoneal dialysis for chronic renal failure. His urine output was recently measured at 110 mL/24 hours. He describes to his physician that he has been experiencing abdominal cramping and his dialysate has been cloudy. Which of the following is the most appropriate empiric regimen for this patient? A.Cefazolin plus ceftazidime (LD 500 mg/L, MD 125 mg/L) B. Cefepime (LD 500 mg/L, MD 125 mg/L) C. Cefazolin plus ceftazidime (LD 625 mg/L, MD 156 mg/L) D.Cefazolin (LD 500 mg/L, MD 125 mg/L) plus gentamicin (LD 8 mg/L, MD 4 mg/L)

A.Cefazolin plus ceftazidime (LD 500 mg/L, MD 125 mg/L)

Which of the following statements regarding nonmotor symptoms associated with PD is true? A.Depression is common in PD and often occurs prior to the appearance of motor symptoms B. Diarrhea is usually problematic throughout the course of PD in most patients C. Sleep problems are related to drug therapy for PD and can easily be treated by changing medication D. Hallucinations rarely need treatment E. The risk of falls in patients with PD is similar to that of non-PD patients

A.Depression is common in PD and often occurs prior to the appearance of motor symptoms

Penetrating abdominal trauma resulting in acute bacterial peritonitis can be treated with oral antimicrobial therapy with the following: A.Levofloxacin plus metronidazole B. Ciprofloxacin C. Levofloxacin plus amoxicillinclavulanate D.Moxifloxacin

A.Levofloxacin plus metronidazole

A patient has been undergoing CAPD and presents with cloudy dialysate, an intermittent, mild fever, and an elevated WBC. The most important factor(s) to consider when selecting an initial antimicrobial agent is (are): A.The dialysis center's and the patient's history of infecting organisms and their sensitivities B. How long the patient has undergone CAPD and the time interval since their last infection C. The extent of the patient's residual renal function D.The modality of dialysis (CAPD or APD)

A.The dialysis center's and the patient's history of infecting organisms and their sensitivities

True or false. Combination antimicrobial regimens after surgical intervention are required for treatment of acute bacterial contamination after abdominal trauma, when the patient is seen within 2 hours of injury. A.True B. False

A.True

*A 60 yo woman is evaluated during a follow-up visit for HTN. History is also notable for HLD. She tolerates her medications well except for minor pedal edema since starting her anti-hypertensive medication. She is active and plays tennis 3x/wk. Current medications are amlodipine 5mg/d and rosuvastatin. On exam, the average of 2 BP readings is 152/86 which is consistent with measurements she has obtained at home for 3 months. HR 64, BMI 22. Trace pedal edema is noted. Labs show normal chemistry panel and urine dipstick shows no protein. Which of the following is the most appropriate next step in management? A: Add Lisinopril B: Add metoprolol C: Increase amlodipine to 10mg/d D: Continue current regimen

A: Add Lisinopril

*Of all adverse drug reactions reported in hospitals, the percentage of those that are either allergic or pseudoallergic is: A. 33% B. 24% C. 10% D. 6% E. 99%

B. 24%

When switching a patient with hypothyroidism from an animal product to synthetic levothyroxine (LT4), by how much should the calculated daily dose of the synthetic LT4 be lowered? A. 0 mcg B. 25 to 50 mcg C. 5 to 10 mcg D. 12 to 20 mcg E. 55 to 65 mcg

B. 25 to 50 mcg

Your institution has recently opened an outpatient osteoporosis management clinic, and you have been charged with the task of identifying patients in your practice who would be candidates for drug therapy. Based on the National Osteoporosis Foundation recommendations, who is a candidate for drug therapy to prevent osteoporotic fracture? A. 68-year-old woman with T-score -2.3 at femoral neck and FRAX 10-year probability of hip fracture 2.3% B. 54-year-old woman with T-score -3.4 at spine and FRAX 10-year probability of major osteoporotic fracture 9% C. 72-year-old man with a T-score -1.3 at femoral neck and FRAX 10-year probability of major hip fracture 2.9% D. 88-year-old woman with T-score -1.9 at femoral neck and FRAX 10-year probability of major osteoporotic fracture 18% E. 96-year-old white woman with physical inactivity, low vitamin D intake, alcohol dependence, and impaired vision

B. 54-year-old woman with T-score -3.4 at spine and FRAX 10-year probability of major osteoporotic fracture 9%

Which patient most likely has streptococcal pharyngitis? A. A 12-month-old boy with a 1-day history of rhinorrhea, vomiting, and a temperature of 99.6?F (37.6 °C) and evidence of pharyngeal erythema upon physical examination B. A 5-year-old girl with a 2-day history of sore throat, fever of 101.5?F (38.6°C), pain upon swallowing, poor oral intake, and tonsillar erythema with exudates C. A 9-year-old girl with a 2-day history of sore throat, pain upon swallowing, postnasal drip, and pharyngeal erythema D. A 21-year-old male college student with a 3-day history of rhinorrhea, 2 days of cough and sore throat, and temperature of 100.1?F (37.8°C)

B. A 5-year-old girl with a 2-day history of sore throat, fever of 101.5?F (38.6°C), pain upon swallowing, poor oral intake, and tonsillar erythema with exudates

Which of the following statements is false regarding the diagnosis of IE? A. All patients with suspected IE should have an echocardiogram performed. B. A definite diagnosis requires a culture from the endocardium. C. A clinical diagnosis is made by applying major and minor criteria. D. Diagnosis is a complicated process requiring assessment of clinical, laboratory, and cardiac imaging.

B. A definite diagnosis requires a culture from the endocardium.

Which of the following statements is false? A. IgE-mediated reactions can occur with aspirin. B. A patient who has experienced an allergic or pseudoallergic reaction to aspirin should never receive aspirin for primary or secondary prevention in coronary artery disease. C. The potential cross-reactivity for IgE-mediated reactions between aspirin and COX-1 inhibiting nonsteroidal antiinflammatory agents is fairly small. D. Cross-reactivity between COX-2 inhibitors and aspirin is rare. E. Chronic idiopathic urticaria is a major risk factor for aspirin-induced pseudoallergic reactions.

B. A patient who has experienced an allergic or pseudoallergic reaction to aspirin should never receive aspirin for primary or secondary prevention in coronary artery disease.

Which of the following patients should be recommended a prophylactic regimen prior to undergoing a major dental procedure involving manipulation of gingival tissue? A. A patient with mitral valve prolapse with regurgitation B. A patient with prosthetic cardiac valves C. A patient with hypertrophic cardiomyopathy D. A patient with rheumatic heart disease

B. A patient with prosthetic cardiac valves

A 42-year-old man presents to the pharmacy with complaints of nasal congestion, thick nasal discharge, cough, and headache for 4 days. He has a past medical history that is significant for seasonal allergies to mold and grass and he has had "sinus infections in the past." What do you recommend for him? A. Refer him to a healthcare provider for an antibiotic prescription B. Acetaminophen and nasal saline spray C. Ibuprofen, guaifenesin, and a cough suppressant D. Intranasal decongestant for up to 7 days; if he is still symptomatic in a week, he should contact a healthcare provider

B. Acetaminophen and nasal saline spray

*TB is a 78-year-old man with a history of vertebral compression fractures, hypertension, osteoarthritis, and depression. Spinal kyphosis is noted on physical exam. Laboratory results show normal testosterone and vitamin D levels. DXA results include a T-score of -2.2 in the left hip and -2.9 in the lumbar spine. His physician would like to initiate treatment for osteoporosis. Which initial therapy is recommended for this patient? A. Calcitonin salmon nasal spray 1 spray daily in alternating nostrils B. Alendronate 70 mg orally once weekly 30 minutes before breakfast C. Teriparatide 20 mcg subcutaneously daily D. Denosumab 60 mg subcutaneously every 6 months E. Ibandronate 150 mg by mouth every month

B. Alendronate 70 mg orally once weekly 30 minutes before breakfast

AJ is a 52-year-old male admitted to the hospital with suspected aspiration pneumonia. His home medications are lisinopril 20 mg once daily and omeprazole 20 mg once daily. What organisms need to be considered as causative agents in AJ? A. S. aureus, E. coli, and viridans Streptococci B. Anaerobes, E. coli, and viridans Streptococci C. S. aureus and anaerobes D. Anaerobes and Streptococcus pneumoniae E. Anaerobes, viridians Streptococci, Streptococcus pneumoniae, E. coli, S. aureus

B. Anaerobes, E. coli, and viridans Streptococci

Risk factors for acute bacterial rhinosinusitis include all of the following except: A. Viral upper respiratory tract infection B. Asthma C. Cigarette smoke exposure D. Perennial allergic rhinitis

B. Asthma

*Which of the following does not describe an epidemiologic issue with community-acquired pneumonia? A. Streptococcus pneumoniae and Mycoplasma pneumoniae are the most prevalent bacterial pathogens. B. CAP caused by CA-MRSA is often a necrotizing disease only in elderly patients. C. Necrotizing CAP caused by CA-MRSA is associated with a high (42%) mortality rate. D. Viruses are the predominant cause of CAP in children. E. All of the above

B. CAP caused by CA-MRSA is often a necrotizing disease only in elderly patients.

Which of the following is incorrect regarding fungal endocarditis? A. It is more common in patients that have received a prolonged course of antibiotics. B. Chance of survival is approximately 85%. C. The most commonly associated pathogens are Candida spp. and Aspergillus spp. D. High-dose therapy and surgery are required.

B. Chance of survival is approximately 85%.

Which of the following conditions should be treated concomitantly if a patent has been diagnosed with gonorrhea? A. Syphilis B. Chlamydia C. Genital warts D. Chancroid

B. Chlamydia

Which one of the following is the preferred firstline drug for a patient infected with S. typhi: A. Ceftriaxone B. Ciprofloxacin C. Erythromycin D. Metronidazole E. Trimethoprimsulfamethoxazole

B. Ciprofloxacin

The previous patient begins to develop symptoms of a UTI (fever, flank pain, elevated white blood count). The appropriate therapy for this patient should now be: A. Ciprofloxacin B. Ciprofloxacin plus catheter removal C. Nitrofurantoin D. Nitrofurantoin plus catheter removal E. Hold antibiotics and remove the catheter if possible

B. Ciprofloxacin plus catheter removal

AV is a 17-month-old girl diagnosed with acute otitis media. She presented to the pediatric clinic today with a fever of 101.4?F (38.6 °C ) and tugging of her left ear. She developed rhinorrhea and nasal congestion approximately 5 days ago, but those symptoms have improved. She attends daycare and has a 4-year-old sister who goes to preschool. This is her first episode of acute otitis media. She did have a urinary tract infection when she was 10 months old, which was treated with antibiotics successfully. What risk factor for bacterial resistance is present in AV? A. Female sex B. Daycare attendance C. Antibiotic therapy 10 months ago D. None of these are risk factors for resistance

B. Daycare attendance

*A 65-year-old patient with Parkinson's disease asks you to explain which surgical procedure could be an option for him. Based on available evidence, which of the following is the best answer at this time? A.Pallidotomy B. Deep-brain stimulation C. Thalamotomy D. Spheramine transplantation E. None of the above are true

B. Deep-brain stimulation

*Which of the following associated symptoms of PD will not improve with lowering the dose of PD medications? A.Psychosis B. Depression C. Orthostatic hypotension D. Sleep disturbances E. Nausea

B. Depression

*In which of the following situations is it most appropriate to use diclofenac 1% gel? A. OA affecting the hip B. Desire to limit the systemic absorption of an NSAID C. As adjunctive treatment to oral diclofenac D. As replacement for previous treatment failure of oral diclofenac E. None of the above are appropriate situations for using diclofenac 1% gel

B. Desire to limit the systemic absorption of an NSAID

Patients with mild or subclinical hypothyroidism should be considered for LT4 therapy if the patient has: A. A family history of thyroid disease B. Elevated LDL cholesterol C. Positive TSHR-AbS antibody D. A history of hypertension E. All of the above

B. Elevated LDL cholesterol

True or false. With the phenomenon of third spacing in initial intraabdominal infections, effective circulating blood volume increases in response to a decrease in cardiac output. A.True B. False

B. False

Capsid Protein detection may be used to diagnose which of the following conditions? A. Syphilis B. Genital warts C. Chancroid D. Herpes simplex virus

B. Genital warts

A 46yearold immunocompromised woman is diagnosed with a perforated diverticulum. Which of the following is the best antimicrobial regimen for this patient? A.Cefazolin B. Imipenemcilastatin C. Cefoxitin D.Metronidazole

B. Imipenemcilastatin

Which medication is FDA approved for use in pediatric patients with Crohn's disease (CD)? A. Budesonide B. Infliximab C. Balsalazide D. Methotrexate

B. Infliximab

The immediate intervention with an anaphylactic reaction in an adult should be: A. IV epinephrine 1 to 4 mcg/min and titrate to response B. Intramuscular epinephrine (1:1000) 0.3 mg then repeat every 5 minutes as needed C. Subcutaneous epinephrine (1:1000) 0.6 mg then repeat every 5 minutes as needed D. Intramuscular epinephrine (1:1000) 0.3 mg then repeat every 15 minutes as needed E. Intramuscular epinephrine (1:10,000) 0.3 mg then repeat every 5 minutes as needed

B. Intramuscular epinephrine (1:1000) 0.3 mg then repeat every 5 minutes as needed

*The TSH is the most useful test for screening and diagnosing thyroid hormone abnormalities and for monitoring therapy of hypothyroidism and hyperthyroidism because: A. The TSH will decrease early in the course of primary hypothyroidism. B. It is a more sensitive biological marker of thyroid function versus serum hormone levels. C. It will normalize 1 or 2 days after effective therapy is begun. D. Serum thyroid hormone level assays are no longer available. E. All of the above

B. It is a more sensitive biological marker of thyroid function versus serum hormone levels.

*An 82-year-old man is diagnosed with early PD. His symptoms are mild constipation and tremor in his right hand that are affecting his quality of life. Which of the following medications would you suggest starting? A.Amantadine B. Levodopa C. Tolcapone D. Entacapone E. Pramipexole

B. Levodopa

Decreases in a patient's glomerular filtration rate can significantly decrease urine concentrations of all of the following antibiotics except: A. Gentamicin B. Levofloxacin C. Minocycline D. Nitrofurantoin E. Trimethoprim-sulfamethoxazole

B. Levofloxacin

JF is a 60-year-old female presents to the emergency room complaining of a productive cough (greenish/yellowish stuff) and chest tightness. PMH: IDDM ?10 years, HTN ? 4 years. Vitals: 101.2oF (38.4 °C), HR 80, 118/86, respiratory rate 18,pulse oximetry (oxygen saturation) 96% (0.94) on room air. Ht. 5'5" (165 cm) Wt. 140 lbs (63.6 kg). Allergies: cephalosporins—hives. Home medications are insulin and atenolol 50 mg qd. Chest x-ray: right mid lobe infiltrate. Her electrocardiogram is normal, renal function is normal (creatinine clearance 75 mL/min [1.25 mL/s]), and WBCs are 11.8 cells/mm3 (11.8 ? 106/L). Which of the following would be the most appropriate empirical therapy for JF's pneumonia? A. Azithromycin 500 mg PO every 24 hours B. Levofloxacin 750 mg PO every 24 hours C. Ceftriaxone 1 g IV plus azithromycin 500 mg IV every 24 hours D. Cefepime 2 g IV plus doxycycline 100 mg IV q12h E. All of the above

B. Levofloxacin 750 mg PO every 24 hours

Patients receiving the enema formulation of mesalamine should be instructed to: A. Use the medication on an as-needed basis only. B. Lie on their left side when administering. C. Retain the contents for no more than 30 minutes. D. Avoid administration at bedtime.

B. Lie on their left side when administering.

Which therapy would be considered first-line for treatment of severe active CD in a hospitalized patient? A. Balsalazide B. Methylprednisolone C. Cyclosporine D. 6-mercaptopurine

B. Methylprednisolone

All of the following therapeutic options may be recommended for second or later recurrences of C. difficile infection (CDI) or for prevention of recurrent CDI episodes except: A. Fidaxomicin B. Metronidazole C. Rifaximin D. Tapered or pulsed oral vancomycin E. Fecal microbiota transplantation

B. Metronidazole

*Which of the following is characteristic of a cluster headache? A.Chronic daily pain B. More common in men C. Onset after the age of 50 D. Dull, band-like frontal pain

B. More common in men

Which of the following fluoroquinolone antibiotics may be least effective for empiric treatment of uncomplicated acute cystitis? A. Levofloxacin B. Moxifloxacin C. Ciprofloxacin D. Norfloxacin E. Ofloxacin

B. Moxifloxacin

*In an otherwise healthy patient with OA of the knee and hip who has had an adequate trial of acetaminophen without satisfactory pain relief, which one of the following treatment options would be the most appropriate next therapy? A. Acetaminophen 1,500 mg every 6 hours B. Naproxen 500 mg every 12 hours C. Diclofenac 1% gel applied every 6 hours as needed to the affected joints D. Oxycodone 20 mg every 4 to 6 hours as needed E. Tramadol 100 mg every 4 hours

B. Naproxen 500 mg every 12 hours

*The preferred treatment option for a recurrent uncomplicated UTI in a 25-year-female patient with a history of UTIs treated with trimethoprim- sulfamethoxazole is: A. Doxycycline B. Nitrofurantoin C. Trimethoprim-sulfamethoxazole D. Levofloxacin E. Amoxicillin-clavulanic acid

B. Nitrofurantoin

Which of the following therapeutic interventions would be most appropriate for the acute management of migraine without aura in a 54-year-old woman recently discharged from the coronary care unit (CCU) after an acute myocardial infarction (AMI)? A.Intranasal sumatriptan B. Oral acetaminophen and hydrocodone C. Oral rizatriptan D. Subcutaneous DHE

B. Oral acetaminophen and hydrocodone

Which of the following antibiotic regimens would be the MOST appropriate for a patient with pseudomonal IE? A. Piperacillin/tazobactam for 4 weeks B. Piperacillin/tazobactam plus gentamicin for 6 weeks C. Imipenem plus gentamicin for 4 weeks D. Piperacillin/tazobactam plus surgery for 2 weeks

B. Piperacillin/tazobactam plus gentamicin for 6 weeks

*You are managing the drug therapy of a 60-year-old female patient diagnosed with PD 2 years ago who has not been on medications previously. She is a piano teacher and now complains of slowness and worsening tremor in her right hand, making it difficult for her to work. Based on this information, what is your recommendation? A.Tolcapone B. Pramipexole C. Amantadine D. Selegiline E. Delay treatment

B. Pramipexole

PM is a 68-year-old woman with a past medical history of hypertension and osteoarthritis who has been told by her primary care physician that she should start alendronate 70 mg orally once weekly for treatment of osteoporosis (T-score of -2.6 at femoral neck). Her other medications include chlorthalidone 25 mg daily and ibuprofen 600 mg TID. She recently read in a women's magazine that medications like alendronate can cause "jaw problems," especially in patients who have dental procedures. She is concerned about starting the medication because she needs to have some dental work done. What factor could increase her risk for osteonecrosis of the jaw (ONJ)? A. Concomitant thiazide diuretic use B. Preexisting dental disease C. Use of oral instead of IV bisphosphonate D. Use of concomitant gastrointestinal toxic drugs E. There are no risk factors that could increase her risk of ONJ

B. Preexisting dental disease

*Which of the following is more commonly associated with UC rather than CD? A. Transmural colonic inflammation B. Proctitis C. Fistula formation D. Confinement to the small intestine

B. Proctitis

Which of the following statements is false regarding the use of glucosamine/chondroitin for OA? A. It may take several weeks for beneficial effects to be experienced. B. Products containing glucosamine sulfate are more efficacious than those containing glucosamine hydrochloride. C. Because glucosamine/chondroitin is regulated as a dietary supplement, product standards are inconsistent. D. Patients with a shellfish allergy should exercise caution when selecting chondroitin products. E. All of the above statements are true.

B. Products containing glucosamine sulfate are more efficacious than those containing glucosamine hydrochloride.

PG is a 68-year-old postmenopausal woman with a history of type 2 diabetes mellitus, coronary artery disease, dyslipidemia, GERD, osteoarthritis of the knees, and stage 3 chronic kidney disease (CrCl 32 mL/min [0.53 mL/s]). A recent EGD showed no abnormalities of the upper GI tract. A recent DXA scan showed a T-score of -3 in the left hip. Which one of the following factors precludes the use of bisphosphonate therapy in PG? A. Age B. Renal insufficiency C. GERD D. Osteoarthritis E. Diabetes mellitus

B. Renal insufficiency

DR is a 72-year-old male who presented to the hospital for a hernia repair. PMH is significant for smoking for the last 20 years. He has been intubated for the last 2 days, and attempts to wean DR off of the ventilator failed. The nurses noted increased volume and purulence of secretions from the ventilator. Chest x-ray: left lower lobe infiltrate. Urinary DFA for Legionella is negative. Ht: 5'10" (178 cm) Wt: 75 kg and he has NKDA. PMH is negative, and home medications are 1 multivitamin once daily. Creatinine clearance is 70 mL/min (1.17 mL/s), WBCs are 13.5 cells/mm3 (13.5 ? 106/L), and temperature is 100.4oF (38.0 oC). Which of the following organisms are the most likely pathogens for DR's pneumonia? A. MRSA, extended-spectrum ß-lactamase-producing K. pneumoniae, P. aeruginosa, Acinetobacter spp. B. S. pneumoniae, MSSA, E. coli, K. pneumoniae C. Anaerobes, viridans Streptococci, E. coli, K. pneumoniae D. S. pneumoniae, H. influenzae, M. catarrhalis, M. pneumoniae, C. pneumoniae, and Legionella pneumophila E. All of the above

B. S. pneumoniae, MSSA, E. coli, K. pneumoniae

Which of the following statements is true? A. Enterococci are the predominant organism causing IE. B. Staphylococci IE can be acquired both in the community and hospital. C. IVDUs are more likely to develop streptococcal IE. D. PVE is more likely to present as a culture-negative IE due to it being a subacute illness.

B. Staphylococci IE can be acquired both in the community and hospital.

The drug or drug class that is responsible for the most cases of toxic epidermal necrolysis is: A. ß-Lactam antibiotics B. Sulfonamide antibiotics C. Insulin D. Thiazide diuretics E. Sulfonylureas

B. Sulfonamide antibiotics

A 5-year-old boy and his mother present to the pediatrician. The child has a 2-month history of intermittent fevers, joint pain, and tenderness in seven joints and a rash that occasionally develops across his buttocks. The rash is not present when the physician examines him. The child weighs 18.2 kg (40 lb). Which of the following does the patient likely have? A. RA B. Systemic JIA C. Polyarticular JIA D. Pauciarticular JIA E. Osteoarthritis

B. Systemic JIA

BB, a 65-year-old man who recently was diagnosed with PD, comes to the pharmacy with questions regarding initiating therapy with a dopamine agonist. Which of the following is true? A.It is inappropriate in most situations due to lack of efficacy B. The onset of motor fluctuations may be delayed C. Anticholinergic medications should be concurrently started D. Neuroprotective effects of dopamine agonists enhance quality of life E. Neuropsychiatric side effects are rare

B. The onset of motor fluctuations may be delayed

*In patients with secondary peritonitis, bacteria may enter the abdomen via which of the following: A.Through a hemodialysis catheter B. Through the damage done to the GI tract by blunt trauma C. Through the bloodstream when there is damage to the GI tract D.Through a peritoneal dialysis catheter

B. Through the damage done to the GI tract by blunt trauma

Which treatment symptom pair is the best initial treatment for the nonmotor symptoms of PD? A.Treat seborrhea with oral prednisone B. Treat postural hypotension by increasing the salt and water in his diet C. Treat constipation with a stimulant laxative D. Treat hallucinations with haloperidol E. Treat nausea with prochlorperazine

B. Treat postural hypotension by increasing the salt and water in his diet

*Treatment of genital warts includes which of the following? A. Cefazolin B. Trichloroacetic acid C. Doxycycline D. Penicillin

B. Trichloroacetic acid

Which of the following pharmacologic agents is FDA approved for the prophylaxis of migraine? A.Amitriptyline B. Valproic acid C. Lamotrigine D. Carbamazepine

B. Valproic acid

Pick the best choice for treatment of ocular symptoms of allergic rhinitis. A. intranasal cromolyn B. intranasal mometasone C. intranasal ipratropium D. intranasal olopatadine E. oral pseudoephedrine

B. intranasal mometasone

Pick the drug that should generally be used for no more than 3 consecutive days. A. oral pseudoephedrine B. intranasal naphazoline C. intranasal azelastine D. intranasal ipratropium E. intranasal cromolyn

B. intranasal naphazoline

Pick the most appropriate drug to recommend for relief of severe nasal congestion to facilitate better mucosal contact of concurrently administered intranasal fluticasone furoate. A. oral pseudoephedrine B. intranasal oxymetazoline C. intranasal ipratropium D. intranasal olopatadine E. intranasal cromolyn

B. intranasal oxymetazoline

Pick the medication that is most effective for refractory nasal congestion. A. intranasal ipratropium B. intranasal tetrahydrozoline C. oral pseudoephedrine D. intranasal azelastine E. saline nasal irrigation

B. intranasal tetrahydrozoline

*A 51 yo M is evaluated during a follow up visit for management of newly diagnosed hypertension and diabetes. He started a program of lifestyle modifications for his DM, but hasn't started anti-hypertensive medication. He currently takes no meds. On exam, BP 148/92, HR 76, BMI 33. The remainder of the exam was normal. Labs show Cr 1.5, K+4.2, urine dipstick with no hematuria or proteinuria and spot urine protein-creatinine ratio 50mg/g Which of the following is the most appropriate anti-hypertensive treatment for this patient? A: HCTZ B: Lisinopril C: Lisinopril and amlodipine D: Lisinopril and HCTZ E: Lisinopril and losartan

B: Lisinopril

Which one of the following regimens is considered an adequate trial of acetaminophen before assessing treatment success or failure? A. 325 mg every 6 hours for 1 month B. 325 mg every 6 hours for 2 months C. 1,000 mg every 6 hours for 1 month D. 500 mg every 8 hours for 2 months E. 325 mg every 4 to 6 hours for 1 month

C. 1,000 mg every 6 hours for 1 month

After starting LT4 therapy or changing a dose, when should the TSH be checked? A. 2 days B. 2 weeks C. 6 weeks D. 6 months E. 12 months

C. 6 weeks

If a patient has been diagnosed with primary syphilis, after how many month of treatment should a follow-up quantitative nontreponemal titer be ordered? A. 1, 2, and 3 B. 3, 6, and 12 C. 6, 12, and 24 D. 9, 18, and 36

C. 6, 12, and 24

What is the most reasonable treatment for an intraabdominal infection in a patient who is found to have a new abscess formation? A.The current antimicrobial agent B. Adequate fluid resuscitation to minimize endorgan dysfunction C. A & B D.Source control via an interventional radiologic procedure of drainage

C. A & B

Which of the following drug regimens is most appropriate for a patient with chronic renal insufficiency (creatinine clearance 28 mL/min [0.47 mL/s])? A. Nabumetone 50 mg twice daily B. Celecoxib 100 mg twice daily C. Acetaminophen 1,000 mg three times daily D. Tramadol 100 mg three times daily E. Oxycodone 5 mg immediate-release every 4 to 6 hours as needed

C. Acetaminophen 1,000 mg three times daily

Which of the following treatment regimens would be the most appropriate for a patient with Kingella kingae IE? A. Penicillin G 3 MU every 4 hours for 4 weeks B. Cefazolin 1 g every 8 hours for 6 weeks C. Ceftriaxone 2 g every 24 hours for 4 weeks D. Ciprofloxacin 400 mg every 12 hours for 2 weeks

C. Ceftriaxone 2 g every 24 hours for 4 weeks

*A 7-year-old girl is diagnosed with streptococcal pharyngitis. She developed a non-urticarial rash after receiving amoxicillin/clavulanate for sinusitis last year. What antibiotic regimen is most appropriate for this child? A. Penicillin VK for 10 days B. Azithromycin for 5 days C. Cephalexin for 10 days D. Trimethoprim-sulfamethoxazole for 10 days

C. Cephalexin for 10 days

In order to be diagnosed with PID, which of the following may be included? A. Oral temperature less than 100 °F (less than 37.8 °C) B. Normal C-reactive protein but decreased erythrocyte sedimentation rate C. Cervical infection with N. gonorrhoeae D. Absence of a vaginal discharge

C. Cervical infection with N. gonorrhoeae

A 26-year-old man with a fever of 39.8°C and flank pain who had a renal transplant 6 months ago and is still on high doses of immunosuppressive therapy. Blood cultures are no growth at 48 hours, but Klebsiella pneumoniae (more than 200 CFU/mL[200 x 103CFU/L] ) is isolated from his clean-catch urine sample. The patient has a recent history of a long hospital and intensive care unit stay. His creatinine clearance is currently estimated to be 50 mL/min(0.83 mL/s), and is hepatic function is normal. Based on his medical history, which of the following is the most appropriate empiric antimicrobial therapy? A. Cefazolin B. Cefepime C. Ciprofloxacin D. Doripenem E. No treatment is recommended at this time

C. Ciprofloxacin

The urine identification and susceptibility results from the previous patient return Klebsiella pneumoniae that is sensitive to all of the antibiotics listed in question 8. Which of the following antibiotics is most appropriate for this patient's complicated UTI as outpatient treatment? A. Cefazolin B. Cefepime C. Ciprofloxacin D. Doripenem E. No treatment is recommended at this time

C. Ciprofloxacin

The main reason why combination therapy such as tigecycline is added to ciprofloxacin for secondary peritonitis treatment is due to: A.Tigecycline lacks Enterococcal activity. B. Ciprofloxacin has poor coverage for atypical organisms. C. Ciprofloxacin augments tigecycline for Pseudomonal coverage. D.Both have excellent MRSA coverage, which is required for treatment.

C. Ciprofloxacin augments tigecycline for Pseudomonal coverage.

Which of the following would be the MOST appropriate oral prophylactic antibiotic for a patient who is allergic to penicillin? A. Amoxicillin B. Ceftriaxone C. Clindamycin D. Vancomycin

C. Clindamycin

Autonomic symptoms may occur prior to the motor symptoms of PD. The most common autonomic problem occurring in more than 50% of PD patients is: A.Seborrhea B. Decreased blood pressure C. Constipation D. Diarrhea E. Nausea

C. Constipation

A 2-month-old male child is brought to the hospital 2 days after onset of a very watery diarrhea. His weight is noted to be 12.5 pounds (5.7 kg), which is decreased from his previous recorded weight of 15 pounds (6.8 kg). His blood pressure is low and his heart rate is increased. His fontanelle and eye orbits are noted to be deeply sunken and tears are absent. His extremities are cool and mottled, and his urine output is less than 1 mL/kg/hr. He is noted to be lethargic upon presentation. Which one of the following therapies would be most appropriate for this patient: A. ORT at 50 mL/kg over 2-4 hours B. ORT at 100 mL/kg over 2-4 hours C. D5W at 100 mL/hr for 2 hours, then reassess fluid status D. Lactated Ringer's at 250 mL/hr for at least 6 hours E. Lactated Ringer's at 20 mL/kg over 15-20 minutes and repeat as necessary

C. D5W at 100 mL/hr for 2 hours, then reassess fluid status

Which statement regarding drug desensitization is false? A. Constant daily administration is necessary once the desired dose is reached. B. Severe reactions such as toxic epidermal necrolysis are absolute contraindications for desensitization. C. Desensitization procedures rarely produce any symptoms if performed properly. D. Only type I IgE-mediated allergy may be treated by classical desensitization. E. Desensitization should only be undertaken by a physician with suitable training and experience.

C. Desensitization procedures rarely produce any symptoms if performed properly.

KM is a 66-year-old woman with osteoporosis, hypothyroidism, and osteoarthritis who has been receiving teriparatide 20 mcg subcutaneously once daily for the past 2 years. She has taken no other therapies for osteoporosis. Her baseline T-scores 2 years ago in the femoral neck and spine were -3.6 and -2.4, respectively. Her T-scores from last month are -2.9 and -2.2 in the femoral neck and spine, respectively. What is the best osteoporosis regimen for her at this time? A. Continue teriparatide because she has had a good response in BMD B. Discontinue teriparatide; no further therapy needed due to the BMD response C. Discontinue teriparatide; initiate alendronate 70 mg orally once weekly D. Discontinue teriparatide; initiate raloxifene 60 mg orally once daily E. Discontinue teriparatide; initiate ibandronate 3 mg IV push every 3 months

C. Discontinue teriparatide; initiate alendronate 70 mg orally once weekly

What is the most likely pathogen for secondary bacterial peritonitis affecting the liver? A.Pseudomonas species B. Bacteroides species C. E. coli species D.Proteus species

C. E. coli species

*Which mesalamine formulation would be most appropriate for a patient with active ulcerative colitis (UC) whose disease is located in the descending colon and rectum? A. Delayed-release tablet B. Delayed-release capsule C. Enema D. Suppository

C. Enema

*Antimicrobial agents are contraindicated in which of the following infections because they may induce the expression and release of toxin, which may predispose a patient to develop hemolytic uremic syndrome (HUS): A. Cholera B. Cryptosporidiosis C. Enterohemorrhagic E. coli D. Enterotoxigenic E. coli E. Salmonellosis

C. Enterohemorrhagic E. coli

Rifaximin is approved for the treatment of traveler's diarrhea (TD) caused by which one of the following organisms: A. Campylobacter B. Enterohemorrhagic E. coli C. Enterotoxigenic E. coli D. Salmonella E. Shigella

C. Enterotoxigenic E. coli

*Regarding nonpharmacologic therapies, recommendations for prevention of recurrent UTIs in women of childbearing age should not include: A. Cranberry juice intake daily B. Drink liberal amounts of water daily C. Estrogen replacement D. Lactobacillus probiotics product E. Patient education on reducing potential risk factors

C. Estrogen replacement

AV has been vaccinated appropriately according to recommended childhood immunization schedules. The most likely pathogen causing AV's infection is: A. Streptococcus pneumoniae B. Streptococcus pyogenes C. Haemophilus influenzae D. Staphylococcus aureus

C. Haemophilus influenzae

Which of the following tests do not need to be performed to assess eradication of the infection (i.e., cure)? A. Echocardiogram B. Vitals signs C. Hematologic tests D. Blood cultures

C. Hematologic tests

*Which of the following statements regarding treatment with MAO-B inhibitors is true? A.Greatest efficacy has been seen in late-stage PD B. Neuropsychiatric effects such as depression are common and problematic C. Insomnia is more common with selegiline than with rasagiline D. Dyskinesias usually improve when adding a MAO-B inhibitor E. They are proven to be neuroprotective

C. Insomnia is more common with selegiline than with rasagiline

*Which of the following is the most appropriate pharmacologic recommendation to treat severe nausea and vomiting in a headache patient? A.High flow oxygen therapy B. Oral opioid analgesics C. Intramuscular chlorpromazine D. IV dihydroergotamine (DHE)

C. Intramuscular chlorpromazine

Why is methimazole the antithyroid therapy of choice in most patients with Graves' disease? A. It has a shorter half-life than PTU. B. It is renally excreted, so no adjustment is needed for liver disease. C. It causes fewer adverse effects such as hepatotoxicity. D. It blocks the conversion of T4 to T3. E. A and D are true.

C. It causes fewer adverse effects such as hepatotoxicity.

*A 52-year-old woman presents to her primary care physician with complaints of joint pain, fatigue, and morning stiffness. Laboratory tests reveal a slightly elevated erythrocyte sedimentation rate, negative rheumatoid factor, and mild anemia. At this time, the physician does not have enough data to make a diagnosis. Which of the following is the most appropriate option for this patient? A. Watch and wait B. Prednisone 20 mg orally once daily C. Methotrexate 5 mg orally once weekly D. Etanercept 50 mg IV once weekly E. Tocilizumab 8 mg/kg IV every 28 day

C. Methotrexate 5 mg orally once weekly

Which antibiotic may be associated with development of peripheral neuropathy with long-term use in patients with IBD? A. Clindamycin B. Amoxicillin C. Metronidazole D. Rifampin

C. Metronidazole

Factors associated with poor outcomes of RA include: A. Late age of disease onset B. Negative rheumatoid factor C. Multiple (greater than 20) tender and swollen joints D. Absence of extraarticular manifestations E. Normal ESR levels

C. Multiple (greater than 20) tender and swollen joints

The only FDAapproved agent for the treatment of cryptosporidiosis is: A. Azithromycin B. Fidaxomicin C. Nitazoxanide D. Paromomycin E. Rifaximin

C. Nitazoxanide

Which of the following would be considered a primary intraabdominal infection? A. A patient with small bowel obstruction and peritonitis after receiving chemotherapy B. A patient who was knifed in the abdomen with rupture of the intestine C. Peritonitis in a patient undergoing peritoneal dialysis D.A patient who presents with a perforated GI ulcer

C. Peritonitis in a patient undergoing peritoneal dialysis

Which of the following steps should be taken in patients at greater risk for reactions to radiocontrast media? A. Use diuretics to minimize fluid volume B. Use higher osmolar agents C. Pretreat with prednisone and diphenhydramine D. A and C only E. B and C only

C. Pretreat with prednisone and diphenhydramine

Which of the following is not a goal of antibiotic therapy for streptococcal pharyngitis? A. Minimize spread of infection to close contacts B. Prevention of rheumatic fever C. Prevention of postinfectious glomerulonephritis D. Reduce the duration of symptoms

C. Prevention of postinfectious glomerulonephritis

CD is a 66-year-old female admitted to the hospital for respiratory failure. Upon admission she was intubated. She was improving and on day 9 was starting to be weaned off the ventilator. On day 9 her respiratory symptoms were worsening and full ventilator settings were required. She is diagnosed with ventilator-associated pneumonia. Which of the following organisms are most likely associated with her pneumonia? A. Streptococcus pneumoniae, Haemophilus influenzae, MRSA, and MSSA B. Haemophilus influenzae, enteric GNB, anaerobes, and MSSA C. Pseudomonas aeruginosa, Acinetobacter spp., and MRSA D. Streptococcus pneumoniae, Haemophilus influenzae, viridians Streptococci, and MRSA E. Pseudomonas aeruginosa, Acinetobacter spp., anaerobes, and Streptococcus pneumoniae

C. Pseudomonas aeruginosa, Acinetobacter spp., and MRSA

Which test should be performed prior to initiating therapy with certolizumab? A. Echocardiogram B. Magnetic resonance imaging (MRI) C. Purified protein derivative D. Serum calcium

C. Purified protein derivative

Serotonin receptor agonists (triptans) are available in all of the following dosage forms except: A.Intranasal spray B. Orally disintegrating tablet C. Rectal suppository D. Subcutaneous injection

C. Rectal suppository

Which statement is true concerning the treatment of HSV? A. The first episode of HSV is self-limiting and lasts approximately 6 to 7 days. B. Episodic therapy requires the administration of acyclovir for patients who are resistant to famciclovir. C. Reserved agents for HSV include IV foscarnet or topical cidofovir. D. Suppressive therapy requires the administration of IV imiquimod as a first-line agent.

C. Reserved agents for HSV include IV foscarnet or topical cidofovir.

Which of the following triptans should be used cautiously with concurrent migraine prophylaxis with propranolol due to a drug-drug interaction? A.Eletriptan B. Frovatriptan C. Rizatriptan D. Zolmitriptan

C. Rizatriptan

*Which one of the following microorganisms is the leading worldwide cause of childhood gastroenteritis deaths: A. C. difficile B. E. coli C. Rotavirus D. Salmonella E. Shigella

C. Rotavirus

A 20-year-old college sophomore presents to the student health department with the "worst headache of my life." She provides a history of migraine headache treated with triptans. On physical examination, she is running a fever, her neck is stiffened, and a rash is noted. This type of headache disorder is classified as which of the following? A.Treatment emergent B. Primary C. Secondary D. Uncomplicated

C. Secondary

*A headache characterized by mild to moderate pain and accompanied by cranial muscle tenderness, yet not associated with nausea or focal neurologic symptoms, is most likely which of the following? A.Migraine with aura B. Migraine without aura C. Tension-type headache D. Cluster headache

C. Tension-type headache

Budesonide would be most appropriate for use in a patient with Crohn's disease whose when the disease location is confined to the: A. Duodenum. B. Jejunum. C. Terminal ileum. D. Descending cColon.

C. Terminal ileum.

Which of the following is the most appropriate prophylactic migraine therapy for an obese 62-year-old man with type 2 diabetes mellitus, emphysema, and peripheral vascular disease? A.Methysergide B. Propranolol C. Topiramate D. Valproic acid

C. Topiramate

*A 38-year-old woman with a history of medication overuse headache (MOH) due to daily acetaminophen use should receive which of the following prophylactic treatments to assist her in weaning off short-acting analgesics? A.β-Blocker B. Calcium channel blocker C. Tricyclic antidepressant (TCA) D. Triptan

C. Tricyclic antidepressant (TCA)

*Seven days into a 14-day course of sulfamethoxazole/trimethoprim, a patient begins experiencing joint soreness, low-grade fever, and a mild rash. Which type of Gell and Coombs type reaction is occurring? A. Type I B. Type II C. Type III D. Type IVa E. Type IVb

C. Type III

A 66yearold male, who has been in the ICU for 2 weeks, has recently completed a course of broadspectrum antibiotics for ventilatorassociated pneumonia. He has now developed a severe case of diarrhea that is noted to be profuse, watery, and greenish in color. He has a newonset high fever, his white blood cell count has increased to 27,000/mm3 (27 ? 109/L), and his serum creatinine has increased to 1.8 mg/dL (159μmol/L) from a baseline of 1.0 mg/dL (88 μmol/L). According to current clinical practice guidelines, which one of the following therapies should be recommended: A. Metronidazole 500 mg (PO) three times daily B. Metronidazole 500 mg (PO) three times daily + loperamide after each loose stool C. Vancomycin 125 mg (PO) four times daily D. Vancomycin 125 mg (PO) four times daily + loperamide after each loose stool E. Vancomycin 1 gram (IVPB) every 12 hours

C. Vancomycin 125 mg (PO) four times daily

Pick the recommendation for additional chronic therapy of a 68-year-old male patient with symptomatic benign prostatic hyperplasia/hypertrophy (BPH), marginally controlled hypertension, and persistent moderate-severe allergic rhinitis that is refractory to appropriate doses of cetirizine. A. intranasal ipratropium B. oral pseudoephedrine C. intranasal budesonide D. oral diphenhydramine E. intranasal tetrahydrozoline

C. intranasal budesonide

Pick the most effective drug from the choices below, to recommend as an OTC medication for initial therapy of mild, intermittent (probably seasonal) allergic rhinitis in an otherwise healthy 6-year-old girl. A. oral phenylephrine B. intranasal oxymetazoline C. intranasal cromolyn D. oral diphenhydramine E. intranasal ciclesonide

C. intranasal cromolyn

*A 57 yo African American M is evaluated for treatment of newly diagnosed HTN. History is notable for HLD, which is treated with moderate-dose simvastatin. On exam, BP 151/94, HR 72, BMI 28. Remainder of exam is unremarkable. Labs show Cr 1.0, fasting glucose 104, and K+ 4.5. A urine dipstick demonstrates no blood or protein. In addition to recommending lifestyle modifications, which of the following is the most appropriate initial anti-hypertensive therapy for this patient? A: Amlodipine B: Diltiazem C: HCTZ D: Lisinopril

C: HCTZ

What is the starting daily dose of LT4 in a 65 kg (143 lb) young, healthy patient with overt hypothyroidism? A. 25 mcg B. 50 mcg C. 75 mcg D. 100 mcg E. 150 mcg

D. 100 mcg

*Which of the following is least likely to develop IE? A. 24-year-old male who has a history of IVDU B. 63-year-old female with a prosthetic tricuspid valve C. 32-year-old male with blood cultures positive for S. aureus for 4 days D. 41-year-old female recently discharged for a mastectomy

D. 41-year-old female recently discharged for a mastectomy

For which of the following patients would a COX-2 selective agent be most appropriate? A. 58-year-old woman with metabolic syndrome and gastroesophageal reflux disease B. 69-year-old man with a history of H. pylori peptic ulcer disease C. 53-year-old woman with a strong family history of myocardial infarction and previous GI upset with ibuprofen use D. 47-year-old man with a history of GI bleed from previous indomethacin use E. 70-year-old woman with chronic renal insufficiency who takes low-dose aspirin for cardioprotection

D. 47-year-old man with a history of GI bleed from previous indomethacin use

Which dose of metronidazole is most often administered for 7 days to treat trichomoniasis? A. 250 mg PO 2 times daily B. 500 mg PO 2 times daily C. 250 mg PO 3 times daily D. 500 mg PO 3 times daily

D. 500 mg PO 3 times daily

For patients requiring continuous prophylaxis due to recurrent lower tract UTIs, the recommended duration for a prophylaxis course is: A. 2 weeks B. 1 month C. 3 months D. 6 months E. 1 year

D. 6 months

*Abatacept is most appropriate in which of the following patients? A. A 36-year-old woman diagnosed today with mild rheumatoid factor-negative RA B. A 51-year-old man with a 2-year history of mild RA and an inadequate response to leflunomide monotherapy C. A 65-year-old man with a 10-year history of rheumatoid factor-positive RA and a fear of needles/injections D. A 42-year-old woman with a 6-year history of RA and an inadequate response to methotrexate monotherapy and methotrexate/etanercept combination therapy E. A 4-year-old boy with juvenile idiopathic arthritis (JIA) and an inadequate response to NSAIDs

D. A 42-year-old woman with a 6-year history of RA and an inadequate response to methotrexate monotherapy and methotrexate/etanercept combination therapy

Which of the following are disadvantages of opioid therapy for treatment of OA? A. High incidence of adverse events B. Potential for development of dependence and addiction C. Requirement for higher doses when used concomitantly with acetaminophen or NSAIDs D. A and B only E. All of the above

D. A and B only

*Outcome evaluations to be made on your patient with pneumonia include: A. Improvement of symptoms within 48 to 72 hours for CAP, HAP, HCAP, and VAP B. Resolution of symptoms within 48 to 72 hours for CAP, HAP, HCAP, and VAP C. Consideration of noninfectious reasons for persistence of symptoms D. A and C E. All of the above

D. A and C

CC has had PD for 10 years. He takes one carbidopa/levodopa 25/100 mg tablet at 8 AM, 2 PM, and 8 PM, and he reports that the 2 PM dose and 8 PM dose wear off 1 to 2 hours early. Which of the following options would be best to minimize early wearing off symptoms for a patient on levodopa? A.Administer levodopa with food and add a dopamine agonist B. Change to a liquid formulation C. Increase each dosage amount by one tablet D. Add a dose at 6 PM and delay the 8 PM dose to 10 PM E. Reduce the dosage of carbidopa to 10 mg

D. Add a dose at 6 PM and delay the 8 PM dose to 10 PM

A 76-year old-man with PD complains of early morning dystonia and difficulty moving in bed throughout the night. His stiffness and frozen feeling leads to frequent awakenings. He complains that he does not feel rested in the morning. He denies nightmares, hallucinations, or acting out his dreams. He takes immediate-release levodopa/carbidopa 100/25 at 8 AM, 1 PM, and 8 PM. How would you adjust the patient's medications to improve his sleep duration and early morning symptoms? A.Add clonazepam at bedtime B. Add amantadine at bedtime C. Add an antihistamine at bedtime D. Add a dose of levodopa/carbidopa at bedtime E. Add gabapentin at bedtime

D. Add a dose of levodopa/carbidopa at bedtime

*Which condition is considered a contraindication to infliximab therapy? A. Diabetes B. Hypothyroidism C. Dyslipidemia D. Advanced heart failure

D. Advanced heart failure

*A 2-year-old girl's mother calls her healthcare provider's office for advice. Her daughter has had cold symptoms (rhinorrhea, nonproductive cough, and temperature of 100.5?F [38.1°C]) for the past 3 days. She asks what she can give to her to help her feel better so she can go back to daycare. Which of these is the best therapeutic recommendation? A. An over-the-counter antihistamine and acetaminophen B. Dextromethorphan and guaifenesin C. Echinacea D. Air humidification and nasal saline drops

D. Air humidification and nasal saline drops

*The initial diagnostic evaluation of a recurrent headache with transitory neurologic symptoms includes which of the following laboratory tests? A.Thyroid function tests (TFTs) B. Erythrocyte sedimentation rate (ESR) C. Complete blood count (CBC) D. All of the above

D. All of the above

Which of the following nonpharmacologic therapies is/are appropriate recommendation(s) for migraine headache? A.Consistent sleep-wake cycle B. Caffeine avoidance C. Daily physical activity D. All of the above

D. All of the above

A 5-year-old girl with no significant past medical history presents to the pediatrician due to increased urinary incontinence and general complaints of back pain. Recommend the most appropriate empiric antibiotic regimen. A. Doxycycline B. Ertapenem C. Norfloxacin D. Amoxicillin-clavulanic acid E. Tobramycin

D. Amoxicillin-clavulanic acid

Which of the following is true regarding bacterial vaginosis? A. BV is caused by an overgrowth of polymicrobial aerobic organisms. B. BV occurs when vaginal pH is less than 3.5. C. BV infection commonly causes a thick greenish discharge. D. BV infection results in a fishy odor.

D. BV infection results in a fishy odor.

Which of the following assessments is crucial for the diagnosis of IE? A. Vital signs B. Hematologic tests C. Erythrocyte sedimentation rate D. Blood cultures

D. Blood cultures

The pathogenesis of tension-type headache (TTH) is related to which of the following presumed mechanisms? A.Muscle contraction B. Hypoxemia C. Hereditary factors D. CNS pain sensitization

D. CNS pain sensitization

Which of the following agents should be prescribed in MSM with a diagnosis of gonorrhea and a history of recent travel? A. Doxycycline B. Azithromycin C. Penicillin G D. Ceftriaxone

D. Ceftriaxone

DC is a 68-year-old woman who recently started oral bisphosphonate therapy for treatment of osteoporosis. Because she never had any symptoms associated with osteoporosis, she asks you how she will know if the medications are working. Which monitoring plan meets current National Osteoporosis Foundation recommendations for monitoring osteoporosis therapy? A. Annual laboratory test for N-telopeptides B. Quarterly laboratory tests for C-telopeptides to reduce diurnal and seasonal variation C. Semiannual Achilles tendon bone mineral density measurement D. Central DXA of hip and spine every 2 years E. No monitoring is needed

D. Central DXA of hip and spine every 2 years

Which of the following regimens would be the least likely to adequately relieve moderate-to-severe pain related to hip OA? A. Acetaminophen 1,000 mg every 6 hours B. Ibuprofen 600 mg every 8 hours C. Tramadol 100 mg every 6 hours D. Diclofenac 1% gel 4 g applied every 6 hours E. Celecoxib 200 mg once daily

D. Diclofenac 1% gel 4 g applied every 6 hours

TD is a 68-year-old man with several risk factors for osteoporosis, including current cigarette smoking, alcohol consumption of 3 to 5 drinks per day, low body weight, and physical inactivity. Lab tests showed 25-hydroxyvitamin D level of 18 ng/mL (45 nmol/L). What recommendation can be made for TD to decrease his osteoporosis risk? A. Increase dietary intake of fortified milk, egg yolks, and salt-water fish B. Increase sun exposure C. Calcium citrate + D (200 IU) BID + multivitamin (vitamin D 400 IU) daily D. Ergocalciferol 50,000 IU once weekly for 8 weeks; then 2,000 IU daily E. Ergocalciferol 50,000 IU once weekly indefinitely

D. Ergocalciferol 50,000 IU once weekly for 8 weeks; then 2,000 IU daily

*If a patient has been diagnosed with neurosyphilis and pleocytosis is present, how often should the cerebral spinal fluid be reexamined? A. Every 3 months B. Every 4 months C. Every 5 month D. Every 6 months

D. Every 6 months

Which of the following conditions is commonly seen in Graves' disease? A. Portal hypertension B. Renal calculi C. Ataxia D. Exophthalmos E. Bronchospasm

D. Exophthalmos

RW is a 45-year-old African-American woman with an 18 pack-year smoking history who quit 10 years ago and has a BMI of 32 kg/m2. She is currently on a 1-week prednisone taper for an asthma exacerbation and takes lisinopril 20 mg daily for hypertension. Which one of her characteristics is associated with an increased risk of developing osteoporosis? A. African-American ethnicity B. Past cigarette smoker C. Present systemic oral glucocorticoid therapy D. Female sex E. BMI greater than 30 kg/m2

D. Female sex

Which of the following joints are affected most frequently by rheumatoid arthritis? A. Hips B. Knees C. Ankles D. Fingers E. Shoulders

D. Fingers

The process by which drugs are altered after binding to body proteins making them large enough for antigen-presenting cells to recognize is called: A. Agglutination B. Exfoliation C. Antigenation D. Haptenation E. Compliment formation

D. Haptenation

*Appropriate treatment regimens for PID include: A. Oral cefazolin plus doxycycline B. IV macrolides plus doxycycline C. Oral Unasyn plus probenecid D. IV gentamicin plus clindamycin

D. IV gentamicin plus clindamycin

Pick the most accurate statement about intranasal corticosteroids. A. Chronic use of intranasal corticosteroids for allergic rhinitis is associated with frequent systemic side effects. B. Only fluticasone propionate, among the intranasal corticosteroids, is available OTC. C. Mometasone is the only intranasal corticosteroid indicated for children younger than 6 years of age. D. Intranasal corticosteroids are effective for both phases of the allergic response in allergic rhinitis. E. Intranasal corticosteroids are contraindicated in pregnant women.

D. Intranasal corticosteroids are effective for both phases of the allergic response in allergic rhinitis.

HPV results in which of the following symptoms? A. Vaginal pH greater than 4.5 B. Excessive yellow/green discharge C. Fissures D. Keratotic warts

D. Keratotic warts

Natalizumab's mechanism of action can best be characterized as a: A. TNF-a inhibitor B. Purine antagonist C. Folate antagonist D. Leukocyte adhesion inhibitor

D. Leukocyte adhesion inhibitor

A patient who is on a 2week business trip to Mexico develops an acute onset of profuse, watery diarrhea that does not appear to be bloody. He is not experiencing any abdominal cramping and no fever is present. However, the diarrhea is interfering with his scheduled business meetings. Which one of the following therapeutic options should be recommended: A. Doxycycline alone B. Loperamide + doxycycline C. Loperamide alone D. Loperamide + levofloxacin E. Fidaxomicin alone

D. Loperamide + levofloxacin

Which of the following statements regarding resistance issues associated with Streptococcus pneumoniae is false? A. Macrolide/azalide resistance rates are higher than penicillin resistance rates B. Macrolide/azalide and penicillin resistance rates rose dramatically in the 1990s but have been relatively flat in the 2000s. C. Resistance to the respiratory fluoroquinolones are 1% or less D. Macrolide/azalide resistance rates correlate closely with clinical failure rates E. All of the above

D. Macrolide/azalide resistance rates correlate closely with clinical failure rates

*Which one of the following medications would be contraindicated in a pregnant patient with CD? A. Sulfasalazine B. Budesonide C. Infliximab D. Methotrexate

D. Methotrexate

*Pick the most accurate statement about nasal saline irrigation therapy for allergic rhinitis. A. Nasal saline irrigation therapy is contraindicated in the elderly due to their increased incidence of rhinorrhea. B. Iodized sodium chloride (salt) is best to use for nasal saline irrigations because the iodine exerts additional osmotic effects that reduce nasal congestion. C. Hypertonic sodium chloride solutions are more effective for allergic rhinitis than "normal" (0.9%) saline when used as a nasal irrigation. D. Nasal saline irrigation may provide some benefit to any patient with allergic rhinitis. E. Nasal saline irrigation therapy is contraindicated in pregnant women because of the potential for excess sodium absorption, which can predispose to polyhydramnios and pre-eclampsia.

D. Nasal saline irrigation may provide some benefit to any patient with allergic rhinitis.

Which statement is true concerning genital herpes? A. HSV is self-limiting B. HSV-1 is the most common cause of recurrent GH C. A cure was developed for GH D. Nearly all HSV-2 infections are sexually acquired

D. Nearly all HSV-2 infections are sexually acquired

Which of the following prophylactic treatments could potentially lead to the development of the "serotonin syndrome" in a patient prescribed naratriptan for acute migraine pain? A.Timolol B. Valproic acid C. Topiramate D. None of the above

D. None of the above

Adherence to prescribed antibiotic therapy can reduce bacterial resistance. Which antibiotic regimen would you consider using in a 13-year-old boy with acute bacterial rhinosinusitis to promote adherence? A. Levofloxacin 500 mg QD ? 5 days B. Clarithromycin 500 mg BID ? 3 days C. Amoxicillin 500 mg BID ? 3 days D. None of these are appropriate options

D. None of these are appropriate options

*JD is a 13-month-old boy with uncertain acute otitis media and nonsevere illness. He was treated for an ear infection 4 months ago with amoxicillin and developed a rash. Which of the following is the most appropriate treatment approach for JD? A. Amoxicillin-clavulanate B. Azithromycin C. Cefdinir D. Observation option

D. Observation option

The preferred antibiotic regimen in a male patient with pyelonephritis and concomitant urosepsis due to Pseudomonas aeruginosa is: A. Ertapenem B. Amoxicillin-clavulanic acid C. Fosfomycin D. Piperacillin-tazobactam E. Gentamicin

D. Piperacillin-tazobactam

Bacteremia secondary to which of the following gastrointestinal pathogens is considered an AIDSdefining illness: A. Campylobacter B. C. difficile C. Cryptosporidium D. Salmonella E. Shigella

D. Salmonella

All of the following are considered risk factors for OA except: A. Heavy physical activity B. Age C. Obesity D. Socioeconomic status E. Joint trauma

D. Socioeconomic status

A patient is receiving naproxen 500 mg twice daily with benefit for left knee OA. However, the patient's symptoms have recently worsened. Which one of the following statements is true regarding the potential role of intra-articular corticosteroid injections in this patient? A. Pain relief will occur rapidly and can last for up to 6 months. B. Increased blood pressure, weight gain, and edema are common side effects. C. They are useful for polyarticular symptoms due to their systemic mechanism of action. D. The affected joint should be only injected 2 to 3 times per year. E. They cannot be used as adjunctive treatment with oral therapies due to increased risk for side effects.

D. The affected joint should be only injected 2 to 3 times per year.

In the setting of antithyroid therapy, which of the following statements regarding agranulocytosis is NOT true? A. Agranulocytosis occurs in 0.3% of patients. B. Patients may present with fever and a sore throat. C. Monitoring for agranulocytosis is controversial. D. The incidence of agranulocytosis is higher in patients treated with propylthiouracil. E. Agranulocytosis usually occurs within the first 3 months of therapy.

D. The incidence of agranulocytosis is higher in patients treated with propylthiouracil.

Why should critically ill patients with nonthyroidal illness ("euthyroid sick syndrome") and a low serum T4 level not be treated with LT4? A. The alteration in the thyroid axis is an appropriate physiologic response to metabolic stress. B. Liothyronine (T3) is the preferred treatment. C. There is no intravenous form of LT4 to administer in the ICU. D. The low serum T4 improves patient outcomes. E. A and D are true.

D. The low serum T4 improves patient outcomes.

A patient comes into your pharmacy for a "second opinion." He was told that his PD symptoms were mild and he should delay starting medicine. Which of the following are reasons for delaying levodopa treatment? A.Patients should use nonpharmacologic treatments first B. Dopamine agonists are equally effective to levodopa in controlling motor symptoms C. The later levodopa is started, the later depression symptoms start D. The risk of motor fluctuations is greater in patients starting levodopa versus a dopamine agonist E. Starting with a monoamine oxidase (MAO-B) inhibitor decreases the risk of drug interactions

D. The risk of motor fluctuations is greater in patients starting levodopa versus a dopamine agonist

A 43-year-old cosmetologist with rheumatoid factor-positive RA was maintained on methotrexate 12.5 mg orally once weekly for approximately 7 years. Her past medical history is significant for dyslipidemia, type 1 diabetes, and anorexia. Signs and symptoms of disease are emerging, and the addition of etanercept is under consideration. Which of the following is necessary to finalize the treatment decision? A. Oral glucose tolerance test B. Complete blood count C. Liver function tests D. Tuberculin skin test E. Lipid panel

D. Tuberculin skin test

A certain diagnosis of acute otitis media must have evidence of: A. Middle ear effusion only B. Middle ear effusion and symptoms of acute infection C. Symptoms of acute infection and tympanic membrane inflammation D. Tympanic membrane inflammation, middle ear effusion, and symptoms of acute infection

D. Tympanic membrane inflammation, middle ear effusion, and symptoms of acute infection

*Which of the following treatments would be the MOST appropriate empiric treatment regimen for a PVE (prosthetic valve endocarditis) with Staphylococcus aureus? A. Penicillin G plus gentamicin plus rifampin for 6 weeks B. Nafcillin plus gentamicin for 4 weeks plus surgery C. Vancomycin plus gentamicin for 4 weeks plus rifampin for 2 weeks D. Vancomycin plus rifampin for 6 weeks plus gentamicin for 2 weeks

D. Vancomycin plus rifampin for 6 weeks plus gentamicin for 2 weeks

*Which mode of therapy underlies the management of all patients with allergic rhinitis? A. intranasal corticosteroids B. education of the patient and their support system C. immunotherapy D. allergen avoidance E. oral decongestants

D. allergen avoidance

*Pick the category of medication that is the most effective for most symptoms of allergic rhinitis in most patients. A. oral second-generation antihistamine B. intranasal antihistamine C. oral leukotriene receptor antagonist D. intranasal corticosteroid E. oral decongestant

D. intranasal corticosteroid

*Pick the symptom of allergic rhinitis that is usually the most bothersome. A. sneezing B. nasal itch C. chemosis D. nasal congestion E. rhinorrhea

D. nasal congestion

Pick the therapy, from the choices below, which is best limited only to patients with concurrent allergic rhinitis and asthma. A. oral montelukast B. intranasal triamcinolone C. intranasal cromolyn D. subcutaneous omalizumab E. intranasal ipratropium

D. subcutaneous omalizumab

The appropriate duration of treatment for most secondary intraabdominal infections of mild to moderate severity is: A.24 hours B. 10 days C. 3 days D.4 to 7 days

D.4 to 7 days

Drug XX has just been approved by the FDA for treatment of intraabdominal infections. What should be considered by your Pharmacy & Therapeutics Committee before the medication gets added to your hospital formulary? A.Its availability to different dosage forms, including both IV and PO B. Its microbiologic spectrum of activity C. Its cost to the institution and the patient D.All of the above

D.All of the above

*Communityacquired complicated intraabdominal infections can be treated with the following agent(s) for a highseverity infection: A.Ampicillin-sulbactam B. Ticarcillin-clavulanate C. Amoxicillin-clavulanate D.Piperacillin-tazobactan

D.Piperacillin-tazobactan

*A 48 yo woman is evaluated during a follow up visit for newly diagnosed HTN, confirmed by multiple measurements at home and in the office. PMH of HLD for which she is taking atorvastatin. Lifestyle modifications have been recommended. On exam, BP 160/92, HR 64, BMI 32. Remainder of exam unremarkable. Labs show Cr 1.1mg/dL, fasting glucose 114 and K+ 4.0, urine dipstick demonstrates no blood or protein. Which of the following is most likely to be effective in controlling this patient's HTN? A: Amlodipine B: Lisinopril C: Losartan D: Lisinopril and amlodipine E: Losartan and Lisinopril

D: Lisinopril and amlodipine

*A 43 yo M is evaluated during a routine physical examination. He has no current symptoms and no PMH. FH is notable for DM and HTN in 2 1st degree relatives. He takes no meds. On physical exam, initial BP is 144/86; repeat measurement after 5 minutes of rest are 136/86 and 134/88. BMI is 32. The remainder of the exam is normal. Labs show normal Cr and plasma glucose levels. In addition to lifestyle modifications, which of the following is the most appropriate next step in the management of this patient's blood pressure? A: Initiate a low dose ACE inhibitor B: Initiate low dose chlorthalidone C: Order ambulatory BP monitoring D: Recheck blood pressure in 1 year

D: Recheck blood pressure in 1 year

*What is the target TSH range (mIU/L) for patients being treated for hypo- or hyperthyroidism? A. Less than 10 B. 2.5 to 4.5 C. 1.4 to 4.5 D. 4 to 5 E. 0.5 to 2.5

E. 0.5 to 2.5

Which of the following is not an optional dosing interval used in oral aspirin desensitization protocols? A. 1 hour B. 2 hours C. ½ hour D. 3 hours E. 4 hours

E. 4 hours

*Which of the following is a consequence of undertreatment with LT4? A. Hypercholesterolemia B. Cardiovascular disease C. Fatigue D. Infertility E. All of the above

E. All of the above

Components of the recommended strategy for the diagnosis of ventilator-associated pneumonia include: A. Obtaining either a quantitative or semiquantitative culture of a lower respiratory sample B. On days 2 and 3: check culture results and assess clinical response to therapy. C. If the clinical assessment at day 2 or 3 is improvement in the patient and the cultures are negative, then antibiotics should be stopped. D. A and C E. All of the above

E. All of the above

Nonpharmacologic therapies for OA include which of the following? A. Stretching B. Application of heat C. Weight loss D. Occupational therapy E. All of the above

E. All of the above

Which of the following agents has a longer duration of action than levodopa/carbidopa and would be given to PD patients instead of levodopa/carbidopa to minimize the pulsatile change in dopamine concentrations? A.Levodopa/carbidopa CR formulation B. Pramipexole C. Rotigotine D. Ropinirole E. All of the above

E. All of the above

Which of the following is the main cause for thyroid abnormalities during amiodarone therapy? A. The large iodine content B. Inhibition of thyroid peroxidase (thionamide-like effect) C. Blockage of peripheral thyroxine receptors D. Induction of the catabolism of T3 E. All of the above

E. All of the above

*You are a pharmacist involved in bone density screening during a health fair in your community. Several patients present to your booth for a peripheral bone density measurement. Based on the following descriptions and recommendations from the National Osteoporosis Foundation, who should be referred to a healthcare provider for follow-up and evaluation of bone mineral density by central DXA? A. 71-year-old man with low bone mineral density B. 66-year-old woman with low calcium intake and nicotine dependence C. 53-year-old active African-American man receiving chronic glucocorticoid therapy D. 58-year-old woman with a FRAX 10-year probability of hip fracture of 3.2% E. All of these patients meet criteria for screening by central DXA.

E. All of these patients meet criteria for screening by central DXA.

*Which one of the following laboratory tests has high specificity for detecting rheumatoid arthritis, especially during early disease? A. Rheumatoid factor B. Antinuclear antibodies C. Anti-histone antibody D. SCL-70 E. Anti-cyclic citrullinated peptide

E. Anti-cyclic citrullinated peptide

Which one of the following statements is true when comparing COX-2 selective agents to nonselective NSAIDs? A. COX-2 selective agents are more effective in treating OA than nonselective NSAIDs. B. COX-2 selective agents may have a higher risk for GI adverse events than nonselective NSAIDs. C. COX-2 selective agents inhibit prostacyclin, but nonselective NSAIDs do not. D. COX-2 selective agents do not adversely impact renal function, whereas nonselective NSAIDs can adversely impact renal function. E. COX-2 selective agents may have a higher risk for cardiovascular adverse events than nonselective NSAIDs.

E. COX-2 selective agents may have a higher risk for cardiovascular adverse events than nonselective NSAIDs.

*A 42-year-old woman presents to your local community pharmacy with complaints of increasing joint pain. She mentioned that her primary care physician was still trying to determine whether the diagnosis is osteoarthritis (OA) or rheumatoid arthritis (RA). While at the pharmacy to fill a new prescription for celecoxib, she asks you if she should be concerned about rheumatoid arthritis because she knows very little about it. In addition to joint pain and swelling, you describe which one of the following as being associated with rheumatoid arthritis? A. Asthma B. Colon cancer C. Osteodystrophy D. Renal insufficiency E. Cardiovascular disease

E. Cardiovascular disease

Which of the following is least likely to have a cross-allergy with penicillin? A. Meropenem B. Cephalexin C. Amoxicillin D. Cefotetan E. Cefepime

E. Cefepime

*GH is a 58-year-old female who presents to the emergency room complaining of a productive cough (greenish/yellowish stuff) and chest tightness. She states this feels different from her usual cough. Two weeks earlier, she had developed a cold, which was resolving prior to the onset of the current symptoms. For the last week she babysat her 2-year-old grandson who had an ear infection and could not attend his daycare. PMH: COPD ? 6 yrs Allergies: cephalosporins—hives, shortness of breath SH: smoked 2-ppd ? 10 years, quit 2 years ago, lives with her husband (nonsmoker) Vitals: 100.8 °F (38.2 °C), HR 80, 118/86 Pulse Ox: 82% (0.82) on room air PE: Lungs: rales, rhonchi, decreased breath sounds over right mid lobe Chest x-ray: right mid lobe infiltrate Sputum Gram stain: Moderate WBCs, no organisms seen What signs, symptoms, and risk factors does GH have that are associated with community-acquired pneumonia? A. Cough, shortness of breath, difficulty breathing, rales, rhonchi, decreased breath sounds, chest x-ray findings B. Fever, myalgias, mental status changes, cough, low oxygenation, exposure to grandson, chest x-ray findings, chest tightness C. Chest x-ray findings, cough, fever, shortness of breath, rales, rhonchi, decreased breath sounds, chest tightness D. Cough, rales, rhonchi, decreased breath sounds, low oxygenation, chest tightness, fever, exposure to grandson E. Cough, rales, rhonchi, decreased breath sounds, low oxygenation, chest tightness, fever, chest x-ray findings, exposure to grandson

E. Cough, rales, rhonchi, decreased breath sounds, low oxygenation, chest tightness, fever, chest x-ray findings, exposure to grandson

BM is a 77-year-old woman with a past medical history of osteoporosis and new-onset hypothyroidism who recently suffered a vertebral fracture. She presents to her physician with complaints of difficulty swallowing, trouble bending over, and pain. Upon exam, the physician also notices her slouching posture and loss of height. Which complication is likely not caused by her vertebral fracture? A. Loss of height B. Decreased mobility C. Kyphosis D. Pain E. Difficulty swallowing

E. Difficulty swallowing

*Compared with LT4 monotherapy, combined T3-LT4 therapy: A. Causes significantly greater reduction in LDL cholesterol B. Results in a higher incidence of atrial fibrillation C. Significantly improves weight loss in obese patients D. Enhances adherence E. Has no overall benefit on patient outcomes

E. Has no overall benefit on patient outcomes

*The appropriate treatment for an asymptomatic 65-year-old male with an indwelling catheter, moderate renal impairment, and bacteriuria with pan-susceptible Enterobacter cloacae is: A. Trimethoprim-sulfamethoxazole B. Ciprofloxacin C. Ampicillin-sulbactam D. Cefepime E. Hold antibiotics and remove the catheter if possible

E. Hold antibiotics and remove the catheter if possible

A 56-year-old man recently diagnosed with RA has a medical history significant for chronic renal insufficiency, hepatitis, and diabetes. Which one of the following DMARDs should be considered at this time? A. Leflunomide B. Azathioprine C. Methotrexate D. Sulfasalazine E. Hydroxychloroquine

E. Hydroxychloroquine

Which statement is false regarding C. difficile enterocolitis: A. It is caused by a grampositive, sporeforming anaerobic organism. B. A more virulent strain is now associated with many outbreaks. C. Toxin production is essential for disease to occur. D. More than 90% of cases occur during or following antimicrobial therapy. E. Infections only occur in patients who have been recently hospitalized.

E. Infections only occur in patients who have been recently hospitalized.

SF is a 56-year-old woman with a history of rheumatoid factor-negative RA × 10 years of moderate severity with features of poor prognosis, dyslipidemia × 5 years, and hypertension × 5 years. She is a nonsmoker and drinks 2 glasses of wine weekly. She exercises 4 days per week (aerobics and swimming). Her current medications include methotrexate 15 mg once weekly, folic acid 1 mg daily, hydrochlorothiazide 25 mg daily, ibuprofen 800 mg 3 times daily as needed (used approximately once monthly), and prednisone 7.5 mg daily during acute RA flares. Her BP today is 162/94 mm Hg and CRP is 4 mg/L. During a recent visit with the rheumatologist, SF reported a gradual increase in morning stiffness, joint pain, and swelling over the last 2 months. The rheumatologist would like to initiate combination disease-modifying antirheumatic drug (DMARD) therapy to help control her symptoms and prevent disease progression. Which of the following is the most appropriate agent to add to her current regimen? A. Anakinra 100 mg SC daily B. Rituximab 1000 mg IV × 2 doses C. Tocilizumab 8 mg/kg IV every 4 weeks D. Abatacept 750 mg IV at 0, 2 and 4 weeks then every 4 weeks E. Infliximab 3 mg/kg IV at 0, 2, and 6 weeks then every 8 weeks

E. Infliximab 3 mg/kg IV at 0, 2, and 6 weeks then every 8 weeks

*Pseudoallergic drug reactions differ from allergic reactions in that they: A. Are based on the structure of the drug B. Involve the activation of the patient's immune system C. Represent common biological functions such as direct histamine release D. Require the drug to be bound to a protein E. Involve T-cell activation

E. Involve T-cell activation

Which of the following drugs may be used to quickly relieve tremor, palpitations, anxiety, and heat intolerance seen in patients with hyperthyroidism? A. Radioactive iodine B. Propylthiouracil C. Methimazole D. Lithium E. Propranolol

E. Propranolol

NSAID therapy is associated with toxicity in all of the following organ systems except: A. Gastrointestinal B. Central nervous C. Renal D. Hepatic E. Pulmonary

E. Pulmonary

* The cornerstone of treatment for a patient with cholera is: A. Azithromycin B. Glucosebased ORT C. IV 0.9% NaCl D. Metronidazole E. Ricebased ORT

E. Ricebased ORT

Which of the following statements regarding investigational treatment for PD is true? A.A potentially curative agent is in early-phase trials B. Using duodenal levodopa as initial therapy is preferred because it will delay motor fluctuations C. Deep-brain stimulation remains a viable first option for early-stage PD D. There is clear evidence supporting the use of herbal products E. Safinamide is an investigational MAO-B inhibitor and a dopamine reuptake inhibitor

E. Safinamide is an investigational MAO-B inhibitor and a dopamine reuptake inhibitor

*What type(s) of Gell and Coombs reactions can penicillins cause? A. Types I and II only B. Types II and III only C. Types I and III only D. Type I only E. Types I, II, III, and IV

E. Types I, II, III, and IV

A vaccine is available in the United States for which one of the following infections: A. C. difficile enterocolitis B. Enterohemorrhagic E. coli C. Shigellosis D. Traveler's diarrhea E. Typhoid fever

E. Typhoid fever

RS is a 75-year-old woman with newly diagnosed osteoporosis who is being discharged from the hospital after total hip replacement due to hip fracture. She has never taken any medication for osteoporosis. Her history is significant for GERD, DVT associated with oral contraceptive use 20 years ago, and family history of breast cancer. Which therapy may offer the most benefit in this patient? A. Denosumab 60 mg subcutaneously every 6 months B. Calcitonin 1 spray daily in alternating nostrils C. Raloxifene 60 mg orally daily D. Teriparatide 20 mcg subcutaneously daily E. Zoledronic acid 5 mg IV every 12 months

E. Zoledronic acid 5 mg IV every 12 months

A 46-year-old woman has a long-standing history of rheumatoid factor-positive R A. She weighs 58.2 kg (128 lb) and is 165 cm (5 feet 5 inches) tall. Her most recent medication profile includes prednisone 10 mg orally once daily, etanercept 50 mg IV once weekly (replaced infliximab 2 years ago), methotrexate 7.5 mg orally once weekly, calcium 600 mg orally twice daily, vitamin D 400 units orally once daily, and nabumetone 1,000 mg orally once daily as needed. She reports an increased frequency of joint pain, tenderness, and morning stiffness. The decision is made to modify her treatment regimen. Which one of the following is the most appropriate option? A. Increase to etanercept 50 mg IV once daily B. Discontinue etanercept; begin rituximab 1,000 mg IV × 2 doses C. Discontinue etanercept, initiate abatacept 750 mg IV on days 1, 15, and every 28 days thereafter D. Initiate tocilizumab 8 mg/kg IV once monthly No correct answer available.

No correct answer available.


Kaugnay na mga set ng pag-aaral

Brain Structure and Nervous System

View Set

Chapter: 44 Written Communication and Mail

View Set

4.01 workshop: independent, dependent, and noun clauses

View Set